prove that...sin^4α+sin^2αcos^2α=sin^2α​

Answers

Answer 1

Answer:

Note that LHS means left-hand side and RHS means right-hand side.

Step-by-step explanation:

LHS = sin⁴a + sin²a cos²a

= sin²a (sin²a + cos²a)

At this point, you can use the identity sin²a + cos²a = 1,

= sin²a (1)

= sin²a

= RHS (Proved)

Answer 2

Answer:

Step-by-step explanation:

[tex]\Large \boldsymbol{} \sin^4a+\sin^2a \cos^2a=\sin^2a \\\\\sin^2a(\underbrace{\sin^2a+\cos^2a}_1) =\sin^2a \\\\\sin^2a\cdot 1=\sin^2a \\\\\sin^2a=\sin^2a[/tex]


Related Questions

Find the measure of one interior angle for the following regular polygon

Answers

To find the sum of the interior angles you use the formula (n-2)(180), where n is the number of sides. In this case, the number of sides is 12, so you get 1800 degrees. Then you divide by number of angles which is 12. So your answer is 150 degrees.

What is the 6th term of this pattern : 0.1, 0.02, 0.12, 0.14, 0.26, ____?




If you will solve i will give brainliest.

Answers

Answer:

0.28

Step-by-step explanation:

(-9) + (-4) =
(+3) - (-5) =
(+2)x(+2) =
(+36) + (+4)
(+5) X (+7) =
(+8) (+9) =
(-4) + (-2) =
(+3) -(-6) =
(-6)*(-3) =
(-4) + (-4)
(+5) X (+7)
(+4)-(-5) =
(+4) - (+3) =
(+10) + (+5)
(+3) (+8)

Answers

[tex]\\ \bull\sf\longmapsto -9+(-4)=-9-4=-13[/tex]

[tex]\\ \bull\sf\longmapsto +3-(-5)=3+5=8[/tex]

[tex]\\ \bull\sf\longmapsto 2\times 2=4[/tex]

[tex]\\ \bull\sf\longmapsto 36+4=40[/tex]

[tex]\\ \bull\sf\longmapsto 5\times 7=35[/tex]

[tex]\\ \bull\sf\longmapsto 8(9)=72[/tex]

[tex]\\ \bull\sf\longmapsto -4+(-2)=-4-2=-6[/tex]

[tex]\\ \bull\sf\longmapsto 3-(-6)=3+6=9[/tex]

[tex]\\ \bull\sf\longmapsto -6(-3)=18[/tex]

[tex]\\ \bull\sf\longmapsto -4+(-4)=-4-4=-8[/tex]

[tex]\\ \bull\sf\longmapsto 5(7)=35[/tex]

[tex]\\ \bull\sf\longmapsto 4-(-5)=4+5=9[/tex]

[tex]\\ \bull\sf\longmapsto 4-3=1[/tex]

[tex]\\ \bull\sf\longmapsto 10+5=15[/tex]

[tex]\\ \bull\sf\longmapsto 3(8)=24[/tex]

Simplify 2m^2 – 2m + 3m^2

Answers

Answer:

5m^2-2m

Step-by-step explanation:

2m^2-2m + 3m^2

5m^2-2m

Answer:

5m² - 2m

Step-by-step explanation:

Given

2m² - 2m + 3m² ← collect like terms

= (2m² + 3m²) - 2m

= 5m² - 2m

rational or irrational? and why

Answers

Answer:

Irrational

Step-by-step explanation:

If we evaluate the square root, we realize that the number will not be a perfect square.

Since it is not a perfect square, it will be an unending, never-repeating, decimal.

Therefore, it will be irrational.

What is equivalent to 1/6

Answers

Answer:

0.16666...

6/36

2/12

3/18

4/24

5/30

There's a lot of things.

Identify the volume and surface area of the hemisphere in terms of pie.

Answers

Answer:

it's off topic but which grade are you in

Let x be the sum of the following 2000 numbers​

Answers

Answer:

See attached image for the sum computed using wolfram alpha, raw value is: 49382716049382716049382716049382716049382716049382716049382716049382716049382716049382716049382716049382716049382716049382716049382716049382716049382716049382716049382716049382716049382716049382716049382716049382716049382716049382716049382716049382716049382716049382716049382716049382716049382716049382716049382716049382716049382716049382716049382716049382716049382716049382716049382716049382716049382716049382716049382716049382716049382716049382716049382716049382716049382716049382716049382716049382716049382716049382716049382716049382716049382716049382716049382716049382716049382716049382716049382716049382716049382716049382716049382716049382716049382716049382716049382716049382716049382716049382716049382716049382716049382716049382716049382716049382716049382716049382716049382716049382716049382716049382716049382716049382716049382716049382716049382716049382716049382716049382716049382716049382716049382716049382716049382716049382716049382716049382716049382716049382716049382716049382716049382716049382716049382716049382716049382716049382716049382716049382716049382716049382716049382716049382716049382716049382716049382716049382716049382716049382716049382716049382716049382716049382716049382716049382716049382716049382716049382716049382716049382716049382716049382716049382716049382716049382716049382716049382716049382716049382716049382716049382716049382716049382716049382716049382716049382716049382716049382716049382716049382716049382716049382716049382716049382716049382716049382716049382716049382716049382716049382716049382716049382716049382716049382716049382716049382716049382716049382716049382716049382716049382716049382716049382716049382716049382716049382716049382716049382716049382716049382716049382716049382716049382716049382716049382716049382716049382716049382716049382716049382716049382716049382716049382716049382716049382716049382716049382716049382716049382716049382716049382716049382716049382716049382716049382716049382716049382716049382716049382716049382716049382716049382716049382715160

Answer:

Step-by-step explanation:

First, i divide all terms by 4

sequence is 1,11,111,1111,....,1111...1111 (with 2000 digits)

[tex]u_1=1\\\\u_2=11=10+1=\dfrac{10^2-1}{9}\\\\u_3=111=100+10+1=\dfrac{10^3-1}{9}\\\\u_4=1111=1000+100+10+1=\dfrac{10^4-1}{9}\\....\\u_n=\dfrac{10^n-1}{9}\\\\\\\displaystyle \sum_{i=1}^n\ u_1 =\dfrac{10^1-1}{9}+\dfrac{10^2-1}{9}+\dfrac{10^3-1}{9}+...+\dfrac{10^n-1}{9}\\=\dfrac{1}{9} *(10+10^2+10^3+....+10^n+1-1-(1+1+1+1...+1)\ )\\\\=\dfrac{1}{9}*( \frac{10^{n+1}-1}{9} -(n+1)\ )[/tex]

[tex]\boxed{4+44+444+....+444...4=\dfrac{4}{9}*( \dfrac{10^{2001}-1}{9} -2001\ )}\\\\\\\approx{4,9382716049382716049382716049383*10^{1999} }[/tex]

PLS HELP ASAP PLS PLSPLS
What is the explicit formula for this sequence?
6, 2, -2, -6, ...

Answers

Answer:

B

Step-by-step explanation:

this is just common sense. if you would have tried you could have easily answered this yourself.

the sequence starts with 6. that is a1.

so, the formula simply has to add "something" to 6.

therefore, A and D are out.

and then we see immediately that the sequence is built by subtracting 4 (or adding -4) for every step.

so, we need to add multiples of -4.

therefore, only B is correct (C adds multiples of +4).

Please help explanation if possible

Answers

Answer:

(1, - 2 )

Step-by-step explanation:

Given the equations

3x - 4y = 11 → (1)

y + 3x = 1 ( subtract 3x from both sides )

y = 1 - 3x → (2)

Substitute y = 1 - 3x into (1)

3x - 4(1 - 3x) = 11 ( distribute and simplify left side )

3x - 4 + 12x = 11

15x - 4 = 11 ( add 4 to both sides )

15x = 15 ( divide both sides by 15 )

x = 1

Substitute x = 1 into (2) for corresponding value of y

y = 1 - 3(1) = 1 - 3 = - 2

solution is (1, - 2 )

The number of pounds of one-dollar-a-pound
coffee needed to mix with 80 pounds of 70¢ a
pound coffee to make a mixture worth 84¢ a
pound is
(A) 70
(B) 80
(C) 95
(D) 65

Answers

Answer:

A

Step-by-step explanation:

Let's say we need x pounds of one-dollar-a-pound coffee . The coffee must average out to 84 cents a pound, and the formula for average is

sum of cost of coffee / number of pounds of coffee, so we have

0.84 = total cost of coffee / (x+80) . The total cost of coffee can be found to be the sum of the cost of $1 coffee and 70 cent coffee, so we have

0.84 = (cost of $1 coffee + cost of 70 cent coffee) / (x+80)

The cost of $1 coffee can be found by adding $1 for each pound of one dollar coffee, or $1 * x. Similarly, the cost of 70 cent coffee is equal to 0.70 * 80, so we have

0.84 = (1*x+0.7*80)/(x+80)

0.84 = (x+56)/(x+80)

multiply both sides by (x+80) to remove a denominator

0.84(x+80) = x+56

0.84x + 67.2 = x+56

subtract both sides by 56 and 0.84x to isolate the x and its coefficients

11.2 = 0.16 x

divide both sides by 0.16 to isolate x

11.2/0.16 = x = 70

The number of pounds of a constituent in a mixture given the cost of the

mixture and the cost and mass of the other constituent can be calculated

by using an equation to model the system

The correct option for the number of pounds of one-dollar- pound coffee needed is option A

(A) 70 pounds

The procedure for arriving at the correct option is as follows:

The given parameters are;

The cost of the the coffee for which the mass in the mixture is to be determined = One-Dollar a pound = 100 ¢ a pound

The mass of the coffee 70¢ a pound coffee to be mixed = 80 pounds

The cost per pound of the mixture = 84 ¢ a pound

The required parameter;

The number of pounds of the one-dollar-a-pound (100 ¢ a pound) coffee in the mixture

Method:

Let x (pound) represent the number of pounds of the one-dollar-a-pound coffee in the mixture, we have;

Mass of mixture = Mass of the one-dollar-a-pound in the mixture, x + Mass of 70 ¢ a pound in the mixture, 80

∴ Mass of mixture in pounds = x + 80

Cost = Cost per pound × Number of pound

Find solution by applying the equation;

Cost of the constituents = Cost of the mixture

Where;

Cost of the constituents = $1 × x + 70 ¢ × 80 = 100 ¢ × x  + 70 ¢ × 80

Cost of the mixture = 84 ¢ × (x + 80)

Therefore;

100 ¢ × x  + 70 ¢ × 80 = 84 ¢ × (x + 80)

The above can be expressed as 100·x + 70×80 = 84 × (x + 80)

Expanding, evaluating and collecting like terms gives;

100·x + 5,600 = 84·x + 6,720

100·x - 84·x = 6,720 - 5,600 = 1,120

16·x = 1,120

x = 1,120/16 = 70

The number of pounds of one-dollar- pound coffee needed, x = 70 pounds

Learn more about equation modelling here;

https://brainly.com/question/14102741

20 is 15% of what number?
PLEASE ANSWER ILL GIVE BRAINLIESTTTT

Answers

Answer:

133.33

Step-by-step explanation:

Answer:

133 1/3 or 133.33

Step-by-step explanation:

So first let's turn 15% into a decimal. For me personally, its less intimidating that way.

15%=0.15

now, let's say the number we are trying to find is x.

A trick I use for these questions is to look at the words. "Of" is used when something is multiplied. "Is" is used for equal signs. So if 20 is 0.15 of x, our formula is easy to figure out!

20=0.15x

x=133.33333...

x=133 1/3

Can someone help me out

Answers

It equals 5? The markings means it’s equal right?

PLS HELP ME ASAP!!!!

Answers

The y row changes by 3 each spot. The x row changed by 4 each spot. So the slope is 3/4x.

The number seventeen and seven thousand in decimal form is​

Answers

Answer:

17.00 and 17,000.00

this is your answer to ur question

What is the volume of the cone to the nearest whole number?
20,520pi cm3
43,320pi cm3
86,640pi cm3
136,094pi cm3

Answers

C. 86,640pi cm3


Hope this helps

[tex] \large\begin{gathered} {\underline{\boxed{ \rm {\red{Volume \: of \: Cone \: = \: \pi \: {r}^{2} \: \frac{h}{3} }}}}}\end{gathered}[/tex]

r represents radius of cone

h represents height of cone

height of cone is 90 cm

radius of cone is diameter / 2

‌ ⇛76/2 = 38

‌ ⇛radius = 38 cm

Solution

[tex]\bf \Large \longrightarrow \:π \: × \: (38 \: cm)² \: × \: \frac{90 }{3} [/tex]

[tex]\bf \Large \longrightarrow \: \pi \: \times \: \:1444 \: {cm}^{2} \: \times \: \cancel\frac{90}{3} \: \: ^{30 \: cm} [/tex]

⇥ 43,320 π cm³

Option ( B ) is the correct answer

The blue team scored two more than five times the number of points,p, scored by the red team
Write an expression for the problem

Answers

Answer:

2 + 5p

Step-by-step explanation:

Hi there!

Let [tex]p[/tex] equal the number of points scored by the red team.

We're given:

The blue team scored two more than five times the number of points scored by the red team.

⇒ blue team points = 2 + (5 × red team points)

⇒ blue team points = 2 + 5p

I hope this helps!

Which correlation best describes the data below.

Answers

Answer:

Weak negative correlation

Find x.

A. √-2
B. 4
C. √2/2
D. √6/2

Answers

Answer:

sqrt(2)

Step-by-step explanation:

Since this is a right triangle, we can use trig functions

sin theta = opp / hyp

sin 45 = x/2

2 sin 45 =x

2 (sqrt(2)/2) =x

sqrt(2) =x

simplify the following​

Answers

Answer:

1) 11[tex]\sqrt{3}[/tex]

2) 2[tex]\sqrt{2}[/tex]

3) [tex]20\sqrt{3} + 15\sqrt{2}[/tex]

4) [tex]53 + 12\sqrt{10}[/tex]

5) -2

6) [tex]7\sqrt{2} - 5\sqrt{3}[/tex]

Step-by-step explanation:

1) 2[tex]\sqrt{12}[/tex] + 3[tex]\sqrt{48}[/tex] - [tex]\sqrt{75}[/tex]

=(2 × 2[tex]\sqrt{3}[/tex] )+ (3 × 4[tex]\sqrt{3}[/tex]) - 5[tex]\sqrt{3}[/tex]

= 4[tex]\sqrt{3}[/tex] + 12[tex]\sqrt{3}[/tex] - 5[tex]\sqrt{3}[/tex]

= 11[tex]\sqrt{3}[/tex]

2) 4[tex]\sqrt{8}[/tex] -2[tex]\sqrt{98}[/tex] + [tex]\sqrt{128}[/tex]

= (4 × 2[tex]\sqrt{2}[/tex]) - (2 × 7[tex]\sqrt{2}[/tex]) + 8[tex]\sqrt{2}[/tex]

= 8[tex]\sqrt{2}[/tex] - 14[tex]\sqrt{2}[/tex] +8[tex]\sqrt{2}[/tex]

= 2[tex]\sqrt{2}[/tex]

3) 5[tex]\sqrt{12\\}[/tex] - 3[tex]\sqrt{18} + 4 \sqrt{72} +2\sqrt{75}[/tex]

= 5× [tex]2\sqrt{3}[/tex] - 3×[tex]3\sqrt{2}[/tex] + 4×[tex]6\sqrt{2}[/tex] + 2×[tex]5\sqrt{3}[/tex]

= [tex]10\sqrt{3} - 9\sqrt{2} +24\sqrt{2} +10\sqrt{3}[/tex]

= [tex]20\sqrt{3} + 15\sqrt{2}[/tex]

4) [tex](2\sqrt{2} + 3\sqrt{5} )^{2}[/tex]

= [tex]8 + 12\sqrt{10} + 45[/tex]

= [tex]53 + 12\sqrt{10}[/tex]

5) [tex](1+\sqrt{3} ) (1-\sqrt{3} )[/tex]

= [tex]1 - 3[/tex]

= -2

6) [tex](2\sqrt{6} -1) (\sqrt{3} -\sqrt{2} )[/tex]

= [tex]2\sqrt{18}-2\sqrt{12} -\sqrt{3} +\sqrt{2}[/tex]

= 2×[tex]3\sqrt{2}[/tex] - 2×[tex]2\sqrt{3}[/tex] - [tex]\sqrt{3} + \sqrt{2}[/tex]

= [tex]6\sqrt{2} - 4\sqrt{3} -\sqrt{3} +\sqrt{2}[/tex]

= [tex]7\sqrt{2} - 5\sqrt{3}[/tex]

Hope the working out is clear and will help you. :)

Step-by-step explanation:

please I solved the question in the diagram above

can someone please help me solve this?

Answers

Answer:

-15

Step-by-step explanation:

df(x, y) / dy = lim (f(x, y+h) - f(x, y)) / h =

lim (e^x^2 - 15y - 15h - e^x^2 ‐ 15y) / h = lim -15h / h = -15

so, the first derivative of f(x,y) by y is simply a constant : -15

fy (x, y) = -15 for any and every values of x and y.

the fast track would be derivative calculation :

the derivative by y makes x a constant. and every constant is eliminated for derivation.

so, e^x^2 goes away (= 0 in the derivative).

that leaves -15y = -15×y¹. its derivative simply is

1×-15×y⁰ = -15

An exponential function fx) is reflected across the y-axis to create functiong(x). Which is a true statement
regarding fa) and g(x)?
The two functions have no points in common
The two functions have the same initial value
The two function have opposite output values of each other for any given input value
The graph of the two functions would look exactly the same
Intro

Answers

Answer:

The two functions have the same initial value

Convert 13pi/6 to a degree measure

A=390
B=2450.44
C=30
D=780

Answers

Answer:

390 degrees

Step-by-step explanation:

The conversion factor is 180/pi

13 pi /6 * 180/pi

13/6 *180

390

I
7. The total cost to rent a truck is $100 and $0.20 per km.
a. Determine an algebraic model for the relationship between total cost and distance driven. Use C to
represent total cost (S) to rent the truck and d to represent distance driven (km).

Answers

Answer:

C= 100+ d*0.2

Step-by-step explanation:

Give data

Cost of rental = $100

Cost per km= $0.2

Total cost= C

Distance= d

Let us model the expression

Hence the expression is given as

C= 100+ d*0.2

Use the graph to find the constant of proportionality (unit rate)

Answers

Answer:

9) 10

10) 5

Step-by-step explanation:

Write an equation of the line. What is the equation of the line?

Answers

Answer:

y=-1

Step-by-step explanation:

As the line is horizontal in nature and pass through (2,-1), the equation is y=-1

5^-4 over 5^3
simplify
A. 5^7
B. 5^-1
C. 1/5
D. 1/5^7​

Answers

Answer:

D is the correct answer.

a grocery store buys boxes of cereal for $ 2.00 each and sells them for 50% more. what does the grocery store charge its customers for each box of cereal?

Answers

Answer:

$3.00

Step-by-step explanation:

First find the markup

2.00 * 50%

2 * .5

1

Add this to the original price

2+1 =3

The store sells the cereal for $3.00

Answer:

$3

Step-by-step explanation:

The grocery store pays $2.00 for each box of cereal bought.

First, find 50% of the cost of each box:

2.00 x 0.50 = 1.00

Next, add the additional amount to the starting price:

2.00 + 1.00 = $3.00

$3.00 is your answer.

~

Select two choices that are true about the function f(x)=23x+14/x

Answers

Answer:

There is an asymptote at x = 0

There is an asymptote at y = 23

Step-by-step explanation:

Given the function:

(23x+14)/x

Vertical asymptote is gotten by equating the denominator to zero

Since the denominator is x, hence the vertical asymptote is at x = 0. This shows that there is an asymptote at x = 0

Also for the horizontal asymptote, we will take the ratio of the coefficient of the variables in the numerator and denominator

Coefficient of  x at the numerator = 23

Coefficient of x at the denominator = 1

Ratio = 23/1 = 23

This means that there is an asymptote at y = 23

Identify the glide reflection rule in the given figure.

Question 8 options:

Translation: (x,y) → (x – 5,y); Reflection across y-axis


Translation: (x,y) → (x,y – 5); Reflection across y-axis


Translation: (x,y) → (x,y + 5); Reflection across y-axis


Translation: (x,y) → (x,y + 5); Reflection across x-axis

Answers

Answer:

B

Step-by-step explanation:

The shape clearly is reflected across y axis and the x coordinates remain the same. We can see a change in the y coordinates and the shape has shifted 5 units down. Hence (x, y) -> (x, y-5) and then reflection across y axis is the answer

Answer:

B

Step-by-step explanation:

Other Questions
What point lies on the line with point slope equation y-3=4(x+7)? find sine of angle of inclination to the axis is of line y=1/3x3+3 The Area of a room (rectangle) is 320 square feet. If the length is 20 in, what is the perimeter? A compound contains only carbon, hydrogen, and oxygen. Combustion of 65.76 g of the compound yields 96.38 g of CO2 and 39.46 g of H2O. The molar mass of the compound is 90.078 g/mol. 1. Calculate the grams of carbon (C) in 65.76 g of the compound: 2. Calculate the grams of hydrogen (H) in 65.76 g of the compound. 3. Calculate the grams of oxygen (O) in 65.76 g of the compound. Zelina scored 10% higher on her second quiz than on her first quiz. On her third quiz, Zelina scored 20% higher than on her second quiz. Her third quiz score is what percent higher than her first quiz score? Which of the following is an advantage to using equations?A. Every problem needs an equation if you want to solve it.B. You can forget what an equation represents when it is used byitself.C. When given two names for the same quantity, you can use algebrato solve the equation.SUBMIT How LinkedIns weekly invite limits are affecting marketers? You probably noticed that the ages of the volcanoes do not always decrease linearly toward the present location of the hotspot. Explain in 1-2 sentences why you think this is the case. This Venn diagram shows the pizza topping preferences for 9 students.What elements are in A and B? (Look at picture) III. Fill in the blanks in the dialogue below with the appropriate words from the following list. hace qu que su sus A hora llegaste? A las tres de la tarde. Cunto tiempo que esperas a Juan? Hace media hora lo espero, pero ahora voy a llamar a hermana Marta. La he visto esta maana en la biblioteca con amigas. The graph of a quadratic function has x-intercepts of -7 and -1, and passes through the point (-4,36). Determine the equation of the quadratic function in the formf(x) = a(x - m)(x n). Units to be Assigned Costs Kraus Steel Company has two departments, Casting and Rolling. In the Rolling Department, ingots from the Casting Department are rolled into steel sheet. The Rolling Department received 4,000 tons from the Casting Department in October. During October, the Rolling Department completed 3,900 tons, including 200 tons of work in process on October 1. The ending work in process inventory on October 31 was 300 tons. How many tons were started and completed during October? fill in the blank 1 tons pls help i need both blanks the options for the second one is left, right, up, down PLS HURRY ITS TIMED!! HELPPPPPPPPPPPPPPPPPPPPPPPPPP Determine which equations have the same solution set as StartFraction 2 Over 3 EndFraction minus x plus StartFraction 1 Over 6 EndFraction equals 6 x. x + = 6x by recognizing properties, rather than solving. Check all that apply. what is meant by reaction Hey guys good Morning what's upactually In India it's night xDthat's why A rubber band slingshot is drawn back 0.15 m with a force of 28N and shoots 28 g stone the initial speed of the stone if the rubber band is released is Help plz help me help plz Patrice bought a can of soda using the following coins. $20 $20 , $10, $10, $5.how much money did she spend ?A. $45B. $65C. $55 Imagine a continuation of the experiment described above. How might NS3/4A expression impact Sendai virus replication, as measured by the amount of virus produced by the different cells? a. Cells expressing NS3/4A will produce more Sendai virus than cells that do not express NS3/4A. b. Cells expressing NS3/4A will produce less Sendai virus than cells that do not express NS3/4A. c. The two types of cells will produce roughly equal amounts of Sendai virus.